Staffetta algebra

Polinomi, disuguaglianze, numeri complessi, ...
Avatar utente
Anér
Messaggi: 722
Iscritto il: 03 giu 2008, 21:16
Località: Sabaudia

Re: Staffetta algebra

Messaggio da Anér »

Sempre da qui
Anér ha scritto:Se entrambi i polinomi pari-dispari hanno grado >1, il terzo coefficiente del prodotto non può essere dispari perché si ottiene, tramite la formula di Cauchy (era sua? boh), come somma di due dispari e un pari. Insomma, se un polinomio comincia con $ x^n+a_{n-1}x^{n-1}+a_{n-2}x^{n-2} $ e l'altro comincia con $ x^m+b_{m-1}x^{m-1}+b_{m-2}x^{m-2} $ allora il terzo termine del prodotto è $ (a_{n-2}+b_{m-2}+a_{n-1}b_{m-1})x^{m+n-2} $.
Dunque almeno uno dei polinomi ha grado al più 1, ma dato che i polinomi sono monici e non costanti tale polinomio deve essere della forma x-k, ovvero k è radice intera del prodotto dei due polinomi.
Il problema 38 invece è qui
Anér ha scritto:Dato un intero positivo $ k $, diciamo che un polinomio a coefficienti reali è raffinato se tutti i suoi monomi hanno grado multiplo di $ k $.
Dimostrare che per ogni polinomio $ p\in\mathbb{R}[x] $ esiste $ q\in\mathbb{R}[x] $ tale che $ pq $ è raffinato.
Sono il cuoco della nazionale!
Avatar utente
Anér
Messaggi: 722
Iscritto il: 03 giu 2008, 21:16
Località: Sabaudia

Re: Staffetta algebra

Messaggio da Anér »

Da qui la soluzione al problema 38:
patatone ha scritto:Posso scrivere P(x) come prodotto di polinomi a coefficienti reali di grado 1 o 2.
Se per ogni polinomio $k_i(x)$ di grado 1 o 2 esiste $h_i(x)$ tale che $k_ih_i$ è raffinato, allora prendo $q(x)$ uguale al prodotto degli $h_i$ e ho finito.
Per semplicità prendo p(x) monico, tanto con un coefficiente davanti non cambia nulla, in modo che tutti i $k_i$ siano monici:
1) Se $k_i(x)=x-a$ allora prendo $\displaystyle h_i=\frac{x^k-a^k}{x-a}$
2) Se $k_i(x)=x^2+ax+b=(x-\alpha)(x-\overline{\alpha})$ con $\alpha$ complesso, allora prendo $\displaystyle h_i=\frac{(x^k-\alpha^k)(x^k-\overline{\alpha}^k)}{(x-\alpha)(x-\overline{\alpha})}$
Da qui invece il problema 39 e la sua soluzione:
patatone ha scritto:Siano a,b,c reali positivi tali che $abc\le a+b+c$. Dimostrare che:
$a^2+b^2+c^2\ge \sqrt 3abc$
Anér ha scritto:Io cercherei di omogeneizzare la disuguaglianza usando l'ipotesi: la tesi ha grado 2 a sinistra e 3 a destra, mentre l'ipotesi ha grado 3 a sinistra e 1 a destra. È chiaro che se eleviamo a qualche potenza positiva RHS e LHS della tesi (o delle ipotesi) otteniamo un problema equivalente, quindi riscriviamo la tesi come $ (a^2+b^2+c^2)^2\geq 3a^2b^2c^2 $. Ora abbiamo di bello che a sinistra il grado è 4 e a destra è 6, dunque la differenza fa 2 che è proprio la stessa differenza dei gradi delle ipotesi; perciò moltiplichiamo a sinistra per $ abc $ e a destra per $ a+b+c $. In questo modo otteniamo una tesi che da un lato è più forte (perché con questa operazione abbiamo in un certo senso fatto un favore alla parte destra della tesi, moltiplicandola per una quantità maggiore o uguale), dall'altro è omogenea di grado 7. Otteniamo infatti come nuova tesi $ (a^2+b^2+c^2)^2abc\geq 3a^2b^2c^2(a+b+c) $. La fortuna aiuta gli audaci e possiamo semplificare abc da entrambe le parti. La tesi diventa $ (a^2+b^2+c^2)^2\geq 3abc(a+b+c) $ e questa è vera per mille motivi; ad esempio perché
$ (a^2+b^2+c^2)(a^2+b^2+c^2)\overset{QM-AM}{\geq} \frac{(a+b+c)(a+b+c)}{3}\frac{(a+b+c)(a+b+c)}{3}\overset{AM-GM}{\geq} $
$ \geq(a+b+c)3\sqrt[3]{abc}\sqrt[3]{abc}\sqrt[3]{abc}=3(a+b+c)abc $.
Da qui infine il problema 40.
Anér ha scritto:Sia $ S $ un sottoinsieme infinito dei naturali. Diciamo che un polinomio è meraviglioso se tutti i suoi monomi hanno grado appartenente a $ S $. Sia $ p\in\mathbb{R}[x] $ un polinomio a coefficienti reali. Dimostrare che esiste $ q\in\mathbb{R}[x] $ diverso dal polinomio nullo tale $ pq $ è meraviglioso.
Sono il cuoco della nazionale!
Avatar utente
jordan
Messaggi: 3988
Iscritto il: 02 feb 2007, 21:19
Località: Pescara
Contatta:

Soluzione 40

Messaggio da jordan »

Soluzione problema 40:
Federiko ha scritto:Va bene, ci ho pensato ancora ma non sono riuscito ad aggiustare la soluzione elegante. Quindi procedo con quella bovina :)
Notazioni:
$$p(x)=\sum_{i=0}^d a_ix^i \ \ ;\ \ q(x)=\sum_{i=0}^{f}b_ix^i$$
Scelgo $f=deg(q)$ tale che $\displaystyle |\{ x\in S , 0\le x\le f\}|\ge d+1 $. Esiste perché $S$ è infinito.
$$p(x)q(x)=\sum_{i=0}^{d+f}c_ix^i$$
con
$$c_i=\sum_{k=0}^d a_k b_{i-k}$$
(consideriamo per evitare scocciature che i coefficienti con pedice negativo o maggiore del grado sono zero).
Devo imporre $c_i=0 \forall i\in (\{0,1,2,..., d+f\}\setminus S)$. Questo è un sistema omogeneo di equazioni lineari con incognite i $b_i$. Abbiamo quindi $f+1$ incognite e un numero di equazioni $\le f+d+1-(d+1)=f$ , per scelta di $f$. Il numero di equazioni è strettamente minore del numero delle incognite; il sistema è omogeneo, quindi per non so quale teorema (forse Rouché-Capelli) c'è una soluzione non triviale.
The only goal of science is the honor of the human spirit.
Avatar utente
jordan
Messaggi: 3988
Iscritto il: 02 feb 2007, 21:19
Località: Pescara
Contatta:

Problema 41.

Messaggio da jordan »

Da qui il problema 41:
Federiko ha scritto:Ovviamente il mio problema è una delle mie amate disuguaglianze. Non fatevi intimidire dal testo!

Sia $n>0$ un intero e $x_1 ,x_2,...,x_n$ dei reali tali che $x_1\le x_2\le...\le x_n$. Dimostrare che
$$\displaystyle \left(\sum_{i=1}^n\sum_{j=1}^n |x_i-x_j|\right)^2 \le \frac{2(n^2-1)}{3}\sum_{i=1}^n\sum_{j=1}^n(x_i-x_j)^2$$
e trovarne i casi di uguaglianza.

Ammetto che allo stage in Francia ci dissero subito qual era il caso di uguaglianza e ciò mi aiutò tantissimo a risolvere il problema (se non altro mi aiutò a partire). Per fair play scrivo qui sotto nascosto il caso di uguaglianza:
Testo nascosto:
$x_1,x_2,...,x_n \ $ è una progressione aritmetica
paga92aren ha scritto:Non sono certo della dimostrazione, ho fatto conti per troppo tempo...

1) il point of incident si ha quando $x_i=x_j \; \forall i,j\in \mathbb{N}$ o $x_i=ki \;\; \forall i \in \mathbb{N} k \in \mathbb{R}$
2) Cauchy-Schwarz su $x_i-x_j$ e $i-j$ per ogni $i\not =j$
3) da 2 ottengo che $\frac{n^2(n^2-1)}{6}\sum (x_i-x_j)^2 \geq (n\sum(2i-n-1)x_i)^2$
4) RHS$\geq\frac{4}{n^2}(n\sum (2i-n-1)x_i)^2$
5) LHS$=(2\sum(2i-n-1)x_i)^2$
6) metto assieme 4 e 5 (semplificando per $n^2$) e ottengo un'identità che dimostra la tesi

Queste sono le idee che ho usato. Se mi confermate che tutti i passaggi sono veri scrivo la dimostrazione per intero.

EDIT: avevo sbagliato qualche equazione...
[...]
paga92aren ha scritto:Scusa per i conti che hai dovuto fare, ma credevo che li avessi già fatti quando hai risolto il problema...(o lo hai risolto in un'altro modo?).
Per il caso di uguaglianza abbiamo scritto la stessa cosa: $x_i=ki \Longleftrightarrow x_i-x_j=k(i-j)$

Dimostrazione (dove non specificato la sommatoria con un solo indice è da $1$ a $n$, con due indici da $1$ a $n$ con $i\not= j$):
2) C-S $\sum (x_i-x_j)^2 \sum (i-j)^2 \geq \left( \sum (x_i-x_j)(i-j) \right) ^2$
3) $\sum (i-j)^2= \sum (i^2+j^2-2ij) = 2(n-1)\sum i^2 -2 \sum ij$, calcolo a parte la sommatoria $\sum ij = 2 \sum_{i>j} ij =2\sum i \sum_{j=1}^{i-1} j = 2\sum i\frac{i(i-1)}{2}= \sum i^2(i-1)= \sum i^3 -\sum i^2$ quindi sostituendo $\sum (i-j)^2= 2n\sum i^2 + 2\sum i^3= \frac{n^2(n+1)(2n+1)}{3}-\frac{n^2(n+1)^2}{2}=\frac{1}{6}n^2(n+1)(n-1)$
Ora calcolo $\sum (x_i-x_j)(i-j)=\sum ix_i+jx_j-ix_j-jx_i= 2(n-1)\sum ix_i-2\sum ix_j=2(n-1)\sum ix_i-2\sum x_j \sum i=$$=2(n-1)\sum ix_i -\sum x_j(n(n+1)-2j)= 2n\sum ix_i-n(n+1)\sum x_i=n\sum (2i-n-1)x_i$ Quindi sostituendo nel punto 2 ottengo: $\frac{n^2(n^2-1)}{6}\sum (x_i-x_j)^2\geq (n\sum (2i-n-1)x_i)^2$
4) RHS$=\frac{2(nì2-1)}{3}\sum(x_i-x_j)^2=\frac{4}{n^2}\frac{n^2(n^2-1)}{6}\sum(x_i-x_j)^2\geq \frac{4}{n^2}\left( n\sum (2i-n-1)x_i \right)^2=4(\sum (2i-n-1)x_i)^2$
5) LHS$=(2\sum_{i>j}x_i-x_j)^2=(2\sum (i-1)x_i -2\sum (n-i)x_i)^2=(2\sum (2i-n-1)x_i)^2$
6) Per 4 e 5 la tesi è ovvia.
1) Dato che l'unica disuguaglianza che ho usato è C-S che ha come point of incident solo $x_i-x_j=k(i-j)$ allora vale l'uguaglianza solo se vale quella relazione.
Se vale quella relazione la tesi (opportunamente semplificata per $k$) diventa $(\sum (i-j))^2=\frac{2(n^2-1)}{3}\sum(i-j)^2$ che sostituendo le relazioni dimostrate sopra ottengo: $(2\sum (2i-n-1)i)^2=\frac{2(n^2-1)}{3}\frac{n^2(n^2-1)}{6}$ da cui $\frac{2n(n+1)(2n+1)}{3}-n(n+1)^2=\frac{n(n^2-1)}{3}$ che è sempre verificata.
The only goal of science is the honor of the human spirit.
Avatar utente
jordan
Messaggi: 3988
Iscritto il: 02 feb 2007, 21:19
Località: Pescara
Contatta:

Problema 42.

Messaggio da jordan »

Da qui il problema 42:
paga92aren ha scritto:Per ogni $x_1\in [0,1)$, data la successione se$x_n \not=0$ allora $x_{n+1}=\frac{1}{x_n}-\lfloor \frac{1}{x_n}\rfloor \forall n\geq 1$ altrimenti $x_{n+1}=0$, dimostrare che:
$$\sum_{i=1}^n x_i<\sum_{i=1}^n \frac{F_i}{F_{i+1}} $$
dove $F_i$ è la successione di Fibonacci.
spugna ha scritto:(Legenda: $ S_n = $ somma dei primi n termini)

Se i primi n termini sono tutti diversi da 0, possiamo dimostrare per induzione che il valore massimo di $ S_n $ aumenta con l'aumentare di $ x_n $: per $ n=2 $, fissato $ x_2 $, il valore massimo di $ x_1 $ è $ \dfrac{1}{x_2+1} $, da cui $ S_2 \le x_2+\dfrac{1}{x_2+1} $. La funzione al secondo membro è crescente nell'intervallo [0,1), quindi la somma è massima quando $ x_2 $ è massimo. (*)
Passo induttivo: supponendo che il lemma valga per un certo n, fissiamo $ x_{n+1} $: per ipotesi induttiva, per massimizzare $ S_n $ dobbiamo massimizzare $ x_n $, che quindi dovrà valere $ \dfrac{1}{x_{n+1}+1} $. Ora bisogna massimizzare $ S_{n-1} $, quindi $ x_{n-1}=\dfrac{1}{x_n+1} $, e così via. Resta da dimostrare che se aumenta $ x_{n+1} $ aumenta anche il valore massimo di $ S_{n+1} $: consideriamo una sequenza $ y_1,y_2,...y_{n+1} $ analoga a quella in questione, con $ y_{n+1}>x_{n+1} $,da cui segue $ y_n<x_n \Rightarrow y_{n-1}>x_{n-1} \Rightarrow y_{n-2}<x_{n-2} \Rightarrow ... $ Tuttavia, sommando i termini a coppie, si ha (e lo si può dimostrare con la (*)) $ y_{n+1}+y_n>x_{n+1}+x_n $ , $ y_{n-1}+y_{n-2}>x_{n-1}+x_{n-2} $,... A questo punto la somma degli $ y_i $ risulta ovviamente maggiore (se n è pari, $ x_1 $ e $ y_1 $ rimangono soli, ma $ y_1>x_1 $). Ora dobbiamo dimostrare che per ogni $ n \ge 1 $ il valore massimo di $ S_n $ non potrà mai eguagliare la somma al secondo membro, perciò dobbiamo vedere cosa succede se $ x_n \rightarrow 1 $: seguirà
$ x_{n-1} \rightarrow \dfrac{1}{1+1}=\dfrac{1}{2} $

$ x_{n-2} \rightarrow \dfrac{1}{\dfrac{1}{2}+1}=\dfrac{2}{3} $

$ x_{n-3} \rightarrow \dfrac{1}{\dfrac{2}{3}+1}=\dfrac{3}{5} $

Si ottengono dunque le stesse frazioni del secondo membro, ma si tratta, alternativamente, di limiti massimi e minimi: ancora una volta ci salva la (*), infatti si avrà $ x_n<1 \Rightarrow x_n+x_{n-1}<1+\dfrac{1}{2} $ , $ x_{n-2}<\dfrac{2}{3} \Rightarrow x_{n-2}+x_{n-3}<\dfrac{2}{3}+\dfrac{3}{5} $ , eccetera. Sommando membro a membro ottengo la tesi.

Se invece nella successione compaiono termini nulli, chiamo k l'intero tale che $ x_k \neq 0 \wedge x_{k+1}=0 $: per $ n \le k $, la dimostrazione è analoga; per $ n>k $ il secondo membro della tesi aumenta, mentre il primo resta invariato (perchè si aggiungono termini nulli) C.V.D.

Va bene? Comunque gran bel problema!! :wink:
The only goal of science is the honor of the human spirit.
Avatar utente
jordan
Messaggi: 3988
Iscritto il: 02 feb 2007, 21:19
Località: Pescara
Contatta:

Problema 43.

Messaggio da jordan »

Da qui il problema 43:
spugna ha scritto:Data una sequenza $ a_1,a_2,..a_n $ con $ a_i \ge 0 $ tale che $ a_i \le a_{i+1} \le 2a_i $, si dimostri che è possibile scegliere i segni della somma $ s=\pm a_1 \pm a_2...\pm a_n $ in modo che si abbia $ 0 \le s \le a_1 $
Gigi95 ha scritto:Per $ n=1 $ è ovvio.
Suppongo di saperlo fare con ogni sequenza di $ n $ numeri reali.
Prendo una sequenza con $ n+1 $ elementi e dimostro che riesco a farlo anche con quella!

Per ipotesi induttiva io so scegliere i segni di $ s'=\pm a_2\pm a_3\pm\dots\pm a_{n+1} $ in maniera tale che si abbia $ 0\leq s'\leq a_2 $.
Pongo $ s_0=s'-a_1 $, essendo $ s'\leq a_2\leq2a_1 $, ho $ s_0 \leq a_1 $, ma essendo $ s'\geq 0 $, ho $ s_0\geq-a_1 $.
Quindi se $ s_0\geq 0 $ pongo $ s=s_0 $, mentre se $ s_0<0 $ cambio tutti i segni ottenendo $ s=-s_0 $.
In entrambi i casi la tesi è soddisfatta.

Aspetto conferma prima di postare il prossimo problema.
The only goal of science is the honor of the human spirit.
Avatar utente
jordan
Messaggi: 3988
Iscritto il: 02 feb 2007, 21:19
Località: Pescara
Contatta:

Problema 44.

Messaggio da jordan »

Da qui il problema 44:
Gigi95 ha scritto:Problema 44:

Trovare tutte le funzioni $ f:\mathbb{R}\rightarrow\mathbb{R} $ che soddisfano:
i) $ f(xy+f(x))=xf(y)+f(x) $
ii) $ f(1)=1 $

Buon lavoro.
paga92aren ha scritto:Ignoro la seconda condizione e pongo $x=0$ da cui $f(f(0))=f(0)$
Sostituisco $x=f(0) y=0$ da cui $f(f(0))=f(0)^2+f(0)$ che messo a sistema con l'equazione sopra mi dà $f(0)^2=0 \Longleftrightarrow f(0)=0$
Ora pongo $y=0$ e ottengo $f(f(x))=f(x)$ e poi pongo (nell'equazione iniziale) $y=f(x)$ e ottengo $f((x+1)f(x))=(x+1)f(x)$, poi pongo $x=f(x)$ e ottengo $f((y+1)f(x))=(f(y)+1)f(x)$(1), infine pongo nell'equazione 1 $y=x$ e ottengo $(f(x)+1)f(x)=f((x+1)f(x)=(x+1)f(x)$ da cui si deduce che se $f(x)\not =0$ allora $f(x)=x$.
Ipotizzo che $f(a)=0$ allora voglio dimostrare che $a=0$: pongo nell'equazione iniziale $x=a\; y=1$ e ottengo $f(a)=0=af(1)$ (potrei concludere con $f(1)=1$ ma ho detto all'inizio che non usavo la seconda ipotesi).
Quindi ho due casi: se $f(1)\not =0$ allora $a=0$ da cui concludo che $f(x)=x$ per ogni $x$.
Altrimenti $f(1)=0$ quindi pongo in 1 $y=-1$ e ottengo $0=f(x)(f(-1)+1)$ da cui si deduce che $f(x)=0$ per ogni $x\not = -1$ (e si verifica facilmente che $f(-1)=0$) oppure $f(-1)=-1$ Quindi pongo in 1 $x=-1 \; y=1$ e ottengo $f(-2)=f(-1)=-1$ che è assurdo perché avevo dimostrato che $f(x)=0,x$
Quindi le uniche soluzioni sono $f(x)=x$ e $f(x)=0$ (se aggiungo la seconda condizione si elimina la seconda soluzione).
The only goal of science is the honor of the human spirit.
Avatar utente
jordan
Messaggi: 3988
Iscritto il: 02 feb 2007, 21:19
Località: Pescara
Contatta:

Problema45.

Messaggio da jordan »

Da qui il problema 45:
paga92aren ha scritto:Sia $x_0=2$ e $x_{n+1}=4x_n(1-x_n)$ una successione e $f(t)=(2-4t)^2$ una funzione, calcolare il valore di:

$$\frac{\prod_{i=0}^n {f(x_i)}}{x_{n+2}}$$
patatone ha scritto:noto che $\displaystyle f(x_n)\frac {x_{n+1}}{x_{n+2}}=\frac {4(1-2x_n)^2}{4(1-x_{n+1})}=\frac {(1-2x_n)^2}{(1-2x_n)^2}=1$
Adesso scrivo quel prodotto come:
$\displaystyle \frac{\prod_{i=0}^n f(x_i)}{x_{n+2}}=\frac {1}{x_1}\prod_{i=0}^n f(x_i)\frac {x_{i+1}}{x_{i+2}}$
però per quanto detto prima ognuno dei termini di quella produttoria è uguale a 1 quindi:
$\displaystyle \frac{\prod_{i=0}^n f(x_i)}{x_{n+2}}=\frac {1}{x_1}=-\frac 1 8$
aspetto conferma prima di partire col prossimo
The only goal of science is the honor of the human spirit.
Avatar utente
jordan
Messaggi: 3988
Iscritto il: 02 feb 2007, 21:19
Località: Pescara
Contatta:

Problema 46+testo 47

Messaggio da jordan »

Da qui il problema 46:
amatrix92 ha scritto:Siano $x,y,z \in \mathbb R$ tali che $x^2+y^2+z^2 = 1$ determinare il minimo valore di $ xy+yz+zx $

Sono ben accette soluzioni di ogni tipo ma meno sono elementari più devono essere formali :)
Valenash ha scritto:Poniamo $x + y + z = k$, e $xy + yz + xz = j$ ora abbiamo che:
$(x + y + z)^2 = x^2 + y^2 + z^2 + 2xy +2xz +2zy$
E quindi, sapendo che: $x^2 + y^2 + z^2=1$ e $xy + yz + xz = j$,
$(x + y + z)^2 = 1 + 2 \cdot j $
E poi: $k^2 = 1 + 2 \cdot j$ da cui ricaviamo $j$, che è la quantità che vogliamo minimizzare:
$j = \frac {k^2 - 1}{2}$.
Facciamo la derivata di $\frac {k^2 - 1}{2}$ e poniamola uguale a 0 in modo da avere un minimo (se per valori minori di k la derivata è negativa e per valori maggiori è positiva), risulta quindi:
$\frac {2k}{2}=0 -> k=0$
Controlliamo che sia effettivamente un minimo: se $k=-1$, abbiamo: $\frac {2k}{2} = \frac {2 \cdot(-1)}{2} = -1$, ok.
Pertanto: $x + y + z = k = 0$ e $x^2 + y^2 + z^2=1$, se vogliamo avere $xy + yz + xz $ minimo.
Torniamo a sostituire ora nell'equazione $(x + y + z)^2 = x^2 + y^2 + z^2 + 2xy +2xz +2zy$, otteniamo: $ (0) ^2 = 1 + 2 j $ da cui $j = - \frac {1}{2}$.
Concludiamo mostrando un esempio in cui ciò si verifica, ad esempio ponendo $z=0$, $x= \frac {1}{\sqrt2}$ e $y= -\frac {1}{\sqrt2}$, da cui otteniamo $x + y + z = 0$ e $x^2 + y^2 + z^2=1$, inoltre $xy + yz + xz = - \frac {1}{2}$.
Spero vada bene =)

EDIT: mi ero dimenticato un passaggio, ora l'ho aggiunto. Dovrebbe essere completa adesso (e spero anche giusta =P).
Viste che sono trascorse due settimane, ho postato il nuovo problema 47 qui:
jordan ha scritto:Trovare tutti i polinomi f(x) tali che f(x)=n ha almeno una soluzione razionale per ogni intero n>0.
The only goal of science is the honor of the human spirit.
Avatar utente
Anér
Messaggi: 722
Iscritto il: 03 giu 2008, 21:16
Località: Sabaudia

Re: Staffetta algebra

Messaggio da Anér »

Da qui la soluzione del problema 47
Anér ha scritto:Intanto se il grado del polinomio è d e scelgo $q_1, \cdots, q_{d+1}$ razionali tali che $f(q_i)=i\quad \forall 1\leq i \leq d+1$, allora il polinomio ha coefficienti che risolvono un sistema in d+1 equazioni e d+1 incognite a determinante (di Vandermonde) non nullo, dunque risolvono un sistema di equazioni a coefficienti razionali, dunque i coefficienti di f sono razionali.
Supponiamo ora che d sia maggiore di 1. Allora posso moltiplicare f per il minimo comune multiplo M dei denominatori dei suoi coefficienti e ottenere il polinomio Mf che ha almeno una radice razionale per ogni multiplo di M. Tuttavia è ben noto che se q è radice razionale di $Mf(x)-kM$, visto che quest'ultimo è un polinomio a coefficienti interi deve valere che il denominatore di q divide quello del coefficiente direttivo di Mf(x)- km, dunque l'insieme dei possibili valori di q è discreto, anzi due elementi distinti distano sempre almeno in certo valore $\varepsilon$ che poi è il reciproco del coefficiente direttivo.
D'altra parte il polinomio $Mf(x+\varepsilon)-Mf(x)$ è almeno di primo grado, dunque non costante, e perciò il suo limite ad infinito è infinito. Questo significa che prima o poi passando da una frazione di denominatore buono alla successiva il corrispondente valore di f cambia di più di 100M, dunque anche considerando sia cosa succede verso più infinito che cosa succede verso meno infinito, c'è un multiplo di M che viene "scavalcato". Quello che stò cercando di dire e non mi riesce di esprimere è che un polinomio di grado maggiore o uguale a 2 da un certo punto in poi, ovvero in un certo intorno di $+\infty$ e in un certo intorno di $-\infty$, cresce tropo in fretta (oltre, non dimentichiamolo, a diventare monotono), per cui ci sono mupltipli di M che passando da una frazione buona a quella successiva non vengono raggiunti; tali valori sono infiniti e perciò non vengono raggiunti tutti nemmeno nelle vicinanze dello 0. Insomma non esiste un razionale che raggiunge tali valori, per cui il polinomio f non soddisfa le ipotesi.
Se invece f(x)=ax+b con a e b razionali, e a non nullo, allora f(x)=n per $x=\frac{n-b}{a}\in\mathbb{Q}$.
Dunque i polinomi f cercati sono tutti e soli quelli di primo grado a coefficienti razionali (i polinomi costanti ovviamente non funzionano).
Da qui il problema 48
Anér ha scritto:Trovare tutte le funzioni $f\colon \mathbb{R}\rightarrow\mathbb{R}$ tali che per ogni $(x,y)\in \mathbb{R}^2$ si abbia
$f(x^2+yf(x))=xf(x)+f(xy)$
Sono il cuoco della nazionale!
Omar93
Messaggi: 79
Iscritto il: 15 mar 2011, 18:58

Re:

Messaggio da Omar93 »

julio14 ha scritto:Riscrivo come $ $\left(\frac{2}{x^6}+3x^2\right)\left(\frac{2}{y^6}+3y^2\right) $.
Grazie ai moduli e alle potenze pari, wlog $ $x,y>0 $
Ora minimizzo un singolo fattore con AM-GM
$ $\frac{2}{x^6}+3x^2=x^2+x^2+x^2+\frac{2}{x^6}\ge4\sqrt[4]{2} $
Avremo l'uguaglianza per $ $x^2=\frac{2}{x^6}\rightarrow x=\sqrt[8]{2} $
Quindi il valore richiesto è $ $2\sqrt[8]{2} $

p.s. anche se rubata, bella idea! :D
Forse è un pò tardi ,ma non capisco questa cosa di AM-GM .
$ 2^{43 112 609} - 1 $
Avatar utente
Anér
Messaggi: 722
Iscritto il: 03 giu 2008, 21:16
Località: Sabaudia

Re: Staffetta algebra

Messaggio da Anér »

Scusa, a quale problema della staffetta ti riferisci?
Sono il cuoco della nazionale!
Omar93
Messaggi: 79
Iscritto il: 15 mar 2011, 18:58

Re: Staffetta algebra

Messaggio da Omar93 »

Al primo
$ 2^{43 112 609} - 1 $
Mist
Messaggi: 542
Iscritto il: 01 gen 2011, 23:52
Località: Provincia di Milano

Re: Staffetta algebra

Messaggio da Mist »

AM-GM ti dice che $\displaystyle \frac{\sum_{i=1}^{n}a_i}{n} \geq \sqrt[n]{\prod_{i=1}^{n}a_i}$(1). Poni $n=4$ $a_1=a_2=a_3 = x^2$ e $a_4 = \frac{2}{x^6}$. Sostituendo nella (1) ottieni che $\displaystyle \frac{x^2+x^2+x^2+\frac{2}{x^6}}{4} \geq \sqrt[4]{2} \rightarrow x^2+x^2+x^2+\frac{2}{x^6} \geq 4\sqrt[4]{2}$. Chiaro ? :wink:
"Se [...] non avessi amore, non sarei nulla."
1Cor 13:2

"[...] e se io non so pentirmi del passato, la libertà è un sogno"
Soren Kierkegaard, Aut-Aut, Ed. Mondadori, pag. 102
Omar93
Messaggi: 79
Iscritto il: 15 mar 2011, 18:58

Re: Staffetta algebra

Messaggio da Omar93 »

Chiaro ? :wink:
Ora si 8)
$ 2^{43 112 609} - 1 $
Rispondi